Đến nội dung

caubeyeutoan2302

caubeyeutoan2302

Đăng ký: 25-05-2011
Offline Đăng nhập: 18-10-2013 - 10:52
****-

#297114 Một vài bài số học thú vị

Gửi bởi caubeyeutoan2302 trong 28-01-2012 - 19:39

Các bài toán :



Câu 1: Cho $m,n$ là các số nguyên dương thỏa mãn là : $ \frac{m}{n}=1-\frac{1}{2}+\frac{1}{3}-\frac{1}{4}+.........-\frac{1}{1318}+\frac{1}{1319} $

Chứng minh rằng $m$ chia hết cho 1979 :D

Câu 2: Tìm $n$ để ta có $n!=2^{15}.3^6.5^{3}.7^{2}.11.13$ :)

Câu 3: Hãy xác định tất cả các bộ nguyên dương (a,b) sao cho $a^2.b+a+b$ chia hết cho $a.b^2+b+7$


#297110 Dãy dài số nguyên dài nhất

Gửi bởi caubeyeutoan2302 trong 28-01-2012 - 19:25

Bài toán :

Bạn hãy tìm một dãy dài nhất bao gồm các số nguyên dương phân biệt mà con số đầu tiên là số 1 và số cuối cùng có dạng $ 31^a.5^b.1990^c$ sao cho mỗi số chia hết cho số dứng trước nó . Có bao nhiêu dãy có độ dài này ?


Gợi ý : Mỗi số có trong dãy thỏa mãn đề như thế thì phải có nhiều hơn một thừa số nguyên tố đứng trước nó ., từ đó ta phân tích và tìm kết quả .


#296787 ĐỀ THI TUYỂN SINH LỚP CHUYÊN 10 TRƯỜNG PHỔ THÔNG NĂNG KHIẾU ĐHQG TPHCM NĂM HỌ...

Gửi bởi caubeyeutoan2302 trong 27-01-2012 - 11:59

Câu 1 : Theo ý tưởng là dùng tỷ lệ thức sau đó áp dụng vào hệ phương trình là ổn .
Câu 2 : Bắt nguồn từ giả thuyết $ a,b,c $ thuộc đoạn $ [-1;2] $ ta nghĩ đến bất đẳng thức $ (a+1).(a-2) \ge 0 $ , suy ra $ a^2-a-2 \ge 0 $ , làm tương tự với b,c sau đó cộng vế theo vế kết hợp với $ a^2+b^2+c^2=6 $ ta có điều phải chứng minh ; bài bất đẳng thức chứa căn thì đơn giản , áp dụng công thức.
Câu 3: Theo anh , câu này dùng đồng dư , hồi đó học làm bài này cũng được lắm :D
Câu 5: Ý tưởng của nguyenta98 là ổn rồi :)

Đề này cần kiến thức vững và cẩn thận , tuy vậy vẫn không khó bằng đề thi Năng khiếu năm 2010-2011 tiếp theo


#279851 Ảnh thành viên

Gửi bởi caubeyeutoan2302 trong 23-10-2011 - 11:05

Hì , anh Dark cười tươi nhỉ :)
Phương anh đâu rồi , tới cậu post hình lên cho mọi người chiêm ngưỡng đi :">


#279720 Fibonacci-Bài toán rất đẹp

Gửi bởi caubeyeutoan2302 trong 22-10-2011 - 11:30

Một trong những vẻ đẹp của toán học phải kể đến đó là dãy số Fibonaccci và các hệ quả cùng với hàng ngàn bài toán rất khó xuất phát từ dãy Fibonacci.

Bài toán :

Cho dãy số $ {u_n},(n=1,2,3,...........)$ được xác định như sau đây:

$ u_1=1,u_2=2,u_{n+1}=3u_n-u_{n-1} $

Dãy số $v_n,(n=1,2,3,..........)$ được xác định nhờ dãy $ u_n$ như sau :

$ v_n=arccotu_1+arccotu_2+arccotu_3+..........................+arccotu_n $

Hãy chứng minh :


$ arccotu_1-arccotu_2-...................-arccotu_{n+1}=arccotF_{2n+2} $

( Trong đó $ F_k $ là số hạng thứ $ k $ của dãy Fibonacci )




#279719 Đề thi chọn Đội tuyển HSG tỉnh Nghệ An

Gửi bởi caubeyeutoan2302 trong 22-10-2011 - 11:10

Bài 2-Ngày thứ 1:


Bài này tương tự như dạng mình đã từng post lên rồi này . Hướng làm : Đa thức bất khả quy :

Bổ đề : Nếu p là 1 số nguyên tố thì đa thức : $ x^{p-1}+x^{p-2}+........................+x+1$ là một đa thức bất khả quy trên trường $ Q(x) $

Trở lại bài toán và chú ý rằng số 2011 là số nguyên tố ,
Ta có phương trình nghiệm nguyên là : $x^{2010}+x^{2009}+...........................+x+1=y^5-1$
Áp dụng bổ đề ta có VT là một đa thức bất khả quy còn VP lại là một đa thức khả quy do $y^5-1=(y-1)(y^4+y^3+y^2+y+1)$ do thế phương trình đã cho không có nghiệm nguyên dương. :closedeyes:
P/S : Toàn , em còn cách nào khác không , ở trên đây do anh đã bí mấy pp thường quá nên mới liều làm đa thức bất khả quy chứ nếu không làm bằng số học bình thường cho lành :)


#279448 $ S=z+z^2+z^3+..................+z^n+\dfrac{1}{z...

Gửi bởi caubeyeutoan2302 trong 19-10-2011 - 10:17

Đề bài :




Cho số phức $ z $ thỏa mãn $ z + \dfrac{1}{z}=1 $

Hãy tính giá trị $ S=z+z^2+z^3+..................+z^n+\dfrac{1}{z}+\dfrac{1}{z^2}+\dfrac{1}{z^3}+............+\dfrac{1}{z^n} $




#279377 Bài toán nghiệm nguyên

Gửi bởi caubeyeutoan2302 trong 18-10-2011 - 10:16

Bài 1: Hãy tìm nghiệm nguyên của phương trình sau :

$ x^{17}+x^{16}+x^{15}+....................+x=y^5-33$

Bài 2 :Hãy tìm nghiệm nguyên dương của phương trình :


$ (x-1)^2.(x-2)^2.(x-3)^2.(x-4)^2...............(x-n)^2=y^2-3y+1 $ với $n$ là số nguyên dương tùy ý.

Gợi ý : Dùng các tiêu chuẩn và định nghĩa về đa thức bất khả quy như tiêu chuẩn Eisenstein mở rộng và Peron




#279376 Nghiệm nguyên

Gửi bởi caubeyeutoan2302 trong 18-10-2011 - 10:00

Bài này cũng đăng được 2 ngày rồi , thôi để tớ chém nốt , xusint có cách khác hay không nhưng mình nghĩ bài này là hơi khó vơi THCS đấy.

Trước hết xin đưa ra tiêu chuẩn Eistenstein :
Cho $ P(x)=a_n.x^n+ a_{n-1}.x^{n-1}+..............+a_1.x+a_0 $ là một đa thức hệ số nguyên . Giả sử tồn tại số nguyên tố $ p $ sao cho thỏa mãn các diều kiện sau :
1) $a_n $ không chia hết cho $ p $
2) Tất cả hệ số còn lại đều chia hết cho $ p $
3) $ a_0 $ không chia hết cho $p^2$
Khi đó đa thức $ P(x) $ bất khả quy trên trường $ Q(x) $

Trở về bài toán ta có PT nghiệm nguyên là $ x^2+3=y^5-1 $
Nhận thấy vế trái là một đa thức bất khả quy ( Nếu lấy số $ p=3$ )
Còn vế phải là một đa thức khả quy do $ y^5-1=(y-1)(y^4+y^3+y^2+y+1) $
Suy ra phương trình đã cho không có ngiệm nguyên ^^


#279299 Biểu diễn thành tổng số

Gửi bởi caubeyeutoan2302 trong 17-10-2011 - 17:46

Proplem:

Giả sử cho rằng $ m $ biểu diễn thành tổng của 3 số nguyên dương :

$m= a_1+b_1+c_1=a_2+b_2+c_2=.....=a_k+b_k+c_k $ với mọi số $ a_i,b_i,c_i (i=1,2,3,...,k) $ đều khác nhau và trong cách biểu diễn này không còn tổng 3 số nào ( khác $a_i,b_i,c_i $ bằng $m$ )

Chứng minh a) $ k(m) < \dfrac{2m}{9} $ b) $ k(m) \ge [ \dfrac{m}{6} ] $




#279034 Hình giải tích

Gửi bởi caubeyeutoan2302 trong 15-10-2011 - 12:04

Cách của perfectstrong là đúng rồi đấy , cứ suy nghĩ đơn giản thôi , vì đây là $ n $ điểm phân biệt , lấy 2 điểm bất kì A,B ta sẽ có 2 véc tơ thỏa để bài là $ \vec{AB} , \vec {BA} $ , ta có số cách chọn 2 điểm bất kì là $ C_n^2 $ vậy thì số véctơ cần tìm là $ 2.C_n^2 $ :lol:


#279033 Chứng minh , $ P(x) $ không thể nhận các giá trị 1,3,5,7,9 với mọi...

Gửi bởi caubeyeutoan2302 trong 15-10-2011 - 11:56

Proplem :

Giả sử rằng đa thức $ P(x) $ có hệ số nguyên , nhận giá trị bằng $2$ ứng với $4$ giá trị $x$ thuộc $\mathbb{Z}$. Chứng minh rằng $P(x)$ không thể nhận các giá trị $1,3,5,7,9$ với mọi $x$ thuộc $\mathbb{Z}$




#275130 Chứng mình rằng Mary có chiến thuật để bảo đảm thắng cuộc chơi .

Gửi bởi caubeyeutoan2302 trong 04-09-2011 - 10:04

Bài toán:

Alex và Mary thay nhau viết các chữ số $0$ hay $1$ cho đến khi mỗi người viết được $2001$ chữ số. Mary sẽ là người thắng cuộc nếu cô ấy viết được một số trong biểu diễn nhị phân sao cho số đó không thể viết được dưới dạng tổng $2$ số chính phương. Chứng mình rằng Mary có chiến thuật để bảo đảm thắng cuộc chơi.




#275127 Equation

Gửi bởi caubeyeutoan2302 trong 04-09-2011 - 09:58

Bài toán: Cho bộ số $m,n,p \in R$ . Giải phương trình sau :


$\dfrac{x^3+m^3}{(x+m)^3}+\dfrac{x^3+n^3}{(x+n)^3+\dfrac{x^3+p^3}{(x+p)^3}}+\dfrac{3}{2}.\dfrac{x-m}{x+m}.\dfrac{x-n}{x+n}.\dfrac{x-p}{x+p}=\dfrac{3}{2}$

Có vẻ là 1 phương trình quá khổ nhỉ :geq :Rightarrow :Rightarrow


#272480 Chứng minh rằng nếu trong dãy các số thu được có chứa số $1001$ thì...

Gửi bởi caubeyeutoan2302 trong 15-08-2011 - 10:37

Bài toán :

Ta bắt đầu với một số nguyên dương nào đấy , số này được tác động bởi $2$ toán tử sau đây : Tách chữ số hàng đơn vị của nó rồi đem nhân chữ số này cho $4$, đem tích cộng với phần còn lại của số đã cho ( Ví dụ : $1997$ biến thành : $7*4+199=227$) . Thực hiện lặp đi lặp lại toán tử này . Chứng minh rằng nếu trong dãy các số thu được có chứa số $1001$ thì không có số nào trong các số của dãy là số nguyên tố .